6a633513122c33a87f3f294442a7e2231a0bfd69
[course.git] / latex / problems / Serway_and_Jewett_8 / problem31.46.tex
1 \begin{problem*}{31.46}
2 A circular loop of wire of resistance $R=0.500\U{\Ohm}$ and radius
3 $r=8.00\U{cm}$ is in a uniform magnetic field directed out of the page
4 as in Figure~P31.46.  If a clockwise current of $I=2.50\U{mA}$ is
5 induced in the loop, \Part{a} is the magnetic field increasing or
6 decreasing in time?  \Part{b} Find the rate at which the field is
7 changing in time.
8 \begin{center}
9 \begin{asy}
10 import Mechanics;
11 import ElectroMag;
12
13 real r = 1cm;
14 real dr = 6pt;
15
16 Vector B = BField(phi=90);
17 vector_field(width=2.5*r, height=2.5*r, v=B);
18 draw(scale(r)*unitcircle);
19 Distance Dr = Distance((0,0), (r,0), "$r$");  Dr.draw();
20 draw(arc((0,0), r+dr, angle1=10, angle2=-10), CurrentPen, ArcArrow);
21 label("$I$", (r+dr, 0), align=E);
22 \end{asy}
23 \end{center}
24 \end{problem*}
25
26 \begin{solution}
27 \Part{a}
28 A clockwise current induces an inward magnetic field inside the loop,
29 so the external flux must be increasingly out of the page.  Therefore,
30 the magnetic field is \ans{increasing}.
31
32 \Part{b}
33 The induced \EMF\ must be
34 \begin{align}
35   0 &= \EMF - IR \\
36   \EMF &= IR \;,
37 \end{align}
38 This is related to the changing field via magnetic flux.
39 \begin{align}
40   |\EMF| &= |-\deriv{t}{\Phi_B}| = \deriv{t}{AB} = A\deriv{t}{B}
41     = \pi r^2 \deriv{t}{B} \\
42   \deriv{t}{B} &= \frac{\EMF}{\pi r^2} = \frac{IR}{\pi r^2}
43     = \ans{62.2\U{T/s}} \;.
44 \end{align}
45 \end{solution}